LSAT and Law School Admissions Forum

Get expert LSAT preparation and law school admissions advice from PowerScore Test Preparation.

 Johnclem
  • Posts: 122
  • Joined: Dec 31, 2015
|
#30274
Hello powerscore,
I had a bit of trouble with this argument and it took me a really long time to choose the right answer. In particular I had trouble getting rid of B . Could you please check my reasoning and also see if I got rid of B for the right reasons ?

1- people are more likely to get disabled as they get older.
2- 55-64 (4%), 65-74 (2%) 74+(1%)

Conclusion : this discrepancies in percentages is because business are offering more benfifts .


Analysis : this doesn't seem to be solving any discrepancies. If business are offering more benefits then why aren't the older people getting similarly benefits? - perhaps they are not working as much as the younger people ? Initially I was trying to pick on percentages being used to conclude that MORE of something is being offered. As percentages don't tell us anything about numbers. But that got me no where . Then I viewed this question as a causal and went to the choices to hunt for an alternate choice .

A) treatment of disabilities is irrelevant because it doesn't say anything about the discrepancies in percentages .

B) I found this the be so tempting, I thought the discrepancies in the percentages could be because people are getting their benefits from alternate sources. The only reason I didn't choose this was because the author argument could still stand . In that business could still give more benefits even if a few people are getting benefits from the government.

c) medical advances in prolonging life is irrelevant to getting benefits for disability.

d) I don't even know what this is .. but this could be because because income s have changed.

E) correct : offers and alternate reason . The reasons for the discrepancies was not due to business offering more benefits but because people stop getting benefits after a certain age.

Thanks
John
 Adam Tyson
PowerScore Staff
  • PowerScore Staff
  • Posts: 5153
  • Joined: Apr 14, 2011
|
#30383
Good analysis, John - I think you are right on in most regards. I'll add a little more info to help, but it looks like you won't need much.

First, there is an assumption built into the stimulus that, while reasonable in the real world, may not be justified in the LSAT. That assumption is that the oldest folks getting benefits are getting them from jobs that they no longer hold - they are retired, or left work because of their disability. That's why the explanation in the stimulus helps - the younger folks that we looked at (55-64) were more likely to have held the more recent jobs that offered those benefits, because they retired or left work more recently than did the older folks.

Like I said, that's a solid real-world assumption, but not necessarily justified on the LSAT. My first prephrase was that the argument left that out, telling us nothing about how recently our various disabled folks were employed.

Turns out that that was not the issue, though, and your causal approach was the right one to take. What alternate cause might have led to the effect of this steady reduction of benefits as people aged?

The problem with answer B is bigger than the one you outlined. It's not just that businesses could still give benefits, although that is true. It's also that "some people" is too vague. Are the oldest ones getting the gov't benefits? Are the younger ones? Is it across the board? If B said something like "older people get more gov't benefits and employer-provided benefits decrease as gov't benefits increase" then we would be on to something, but that just gives B too much help.

E is the winner for exactly the reason you cited - it's an alternate cause. E is such a clear winner that you should eliminate B on the grounds that, even if it does help do the job of weakening the argument, it doesn't weaken as much as E, so B is not the best answer. That's a great reason to eliminate an answer choice, because we are not asked "which of these weakens the argument" but rather "which of these weakens it the MOST?" Pick the BEST answer, rather than a GOOD answer, and you can't go wrong.

Great job. Keep that up!
 lsatnoobie
  • Posts: 52
  • Joined: Sep 18, 2017
|
#42863
I had a difficult time choosing Answer E because it does not explain why benefits again decrease at 75 y/o. It makes sense that disability benefits would drop from 4% to 2% once someone hits age 66, but why would it drop again in the 75% group? If anything, if people under MOST employers' insurance plans stop receiving payments after 65, then shouldn't the 65-74 group and 75+ group have the same percentage?
 Claire Horan
PowerScore Staff
  • PowerScore Staff
  • Posts: 408
  • Joined: Apr 18, 2016
|
#43221
Hi lsatnoobie,

Go back to the question stem! It asks you to find the statement that, if true, shows why the explanation given in the stimulus is incomplete. It doesn't purport to offer a full explanation of the statistics, but it does show why the explanation given isn't complete (and you include this in your question, so it looks like you understand why).

The takeaway: Always pay close attention to the question stem to make sure you are answering the question that is being asked! Question stems are often similar to each other, but don't take shortcuts—read them carefully!
 Legallyconfused
  • Posts: 19
  • Joined: Oct 03, 2019
|
#72004
Hi there!

I am having a hard time figuring out what kind of question type this is. Is it possibly an explain or weaken?

Thanks! :-D
 Zach Foreman
PowerScore Staff
  • PowerScore Staff
  • Posts: 91
  • Joined: Apr 11, 2019
|
#72037
LC,
It is a weaken. There is no "explain" question type.
We know it is a weaken because it is going from answer choice to stimulus and it is undermining it, so we know it is family 3, which only has one member: weaken.
 bonnie_a
  • Posts: 32
  • Joined: Jun 05, 2021
|
#90393
Hello, I have a question about answer choice A. I believe this is an alternate explanation for the discrepancy so weakens the argument that it has to do with the increased number of jobs offered for benefits. How I interpreted answer choice A was: if the treatment of disabilities of people (including the elderly) is more successful now, then this may lead to a low likelihood of becoming disabled, which may eventually lead to the reduced need to get disability benefits among the elderly. I noticed from previous posts that this answer choice was not relevant but doesn't it provide another reason why it doesn't have to do with the jobs and thus weaken the argument?
User avatar
 evelineliu
PowerScore Staff
  • PowerScore Staff
  • Posts: 91
  • Joined: Sep 06, 2021
|
#90484
Hi Bonnie,

(A) is incorrect because (A)'s assertion that disabilities are treated better now than in the past affects all of East Wendell's people. Even if the improved care explains why the elderly suffer less disability, it would not explain the shrinking proportion phenomenon.

Hope that helps,
Eveline
User avatar
 Adam354
  • Posts: 29
  • Joined: Feb 08, 2022
|
#93936
Correct me if I'm wrong, but, don't all the answers for this question result in "at best incomplete?"
User avatar
 Adam354
  • Posts: 29
  • Joined: Feb 08, 2022
|
#93937
I did notice that if we are to focus on the explanation, and the explanation discusses jobs, that the only answer that discusses jobs is E, the correct answer. Is that coincidental, or a correct way to solve the problem?

Get the most out of your LSAT Prep Plus subscription.

Analyze and track your performance with our Testing and Analytics Package.